Which of these numbers are divisible by 3 - a. 29 b. 16 c. 24 d. 14?

1 Answer

Answer :

Only c. 24 is evenly divisible by 324/3 =- 8

Related questions

Description : 1, 2, 3, 4, 5, 6, 7, 8, 9, 10, 11, 12, 13, 14, 15, 16, 17, 18, 19, 20, 21, 23, 24, 25, 26, 27, 28, 29, 30, 31, 32, 33, 34, 35, 36, 37, 38, 39, 40, 41, 42, 43, 44, 45, 46, 47, 48, 49, 50'. What number is missing? -Riddles

Last Answer : 22

Description : The odd numbers from 1 to 45 which are exactly divisible by 3 are arranged in an ascending order. The number at 6 th position is (A) 18 (B) 24 (C) 33 (D) 36

Last Answer : Answer: C

Description : What least number would be subtracted from 427398 so that the remaining number is divisible by 15? 1. 13 2. 3 3. 16 4. 11 5. 14

Last Answer : Answer- 2 ( 3) Explanation:- On dividing 427398 by 15 we get the remainder 3, so 3 should be subtracted 

Description : How many numbers between 16 to 189 are divisible by 9 but not by 4?

Last Answer : There are 14 such numbers between 16 and 189.

Description : Which article of the constitution prohibits the employment of children bellow 14 years of age in any factory ,mine or other hazardous job- a) Art-14 b) Art-19 c) Art-24 d) Art-29

Last Answer : c) Art-24

Description : A memory management system has 64 pages with 512 bytes page size. Physical memory consists of 32 page frames. Number of bits required in logical and physical address are respectively: (1) 14 and 15 (2) 14 and 29 (3) 15 and 14 (4) 16 and 32

Last Answer : (3) 15 and 14

Description : Two students appeared at an examination. One of them secured 18marks more than the other and his marks was 72% of the sum of their marks. What are the marks obtained by them? a) 12.5,23.3 b) 26.7,16.0 c) 13.3,14.2 d) 11.45, 29.45 e) 29.8,15.4 

Last Answer : Answer: D  Let the marks secured by them be x and (x + 18)  Then sum of their marks = x + (x + 18) = 2x + 18  Given that (x + 18) was 72% of the sum of their marks  =>(x+18) = 72/100(2x+18)  => ... 11x = 126 x = 11.45  Then (x + 18) = 11.45 + 18 = 29.45  Hence their marks are 11.45 and 29.45

Description : The mean monthly temperature and rainfall of a station are given below: Month Temperature ºC Rainfall mm January 24 2 February 25 1 March 27 0 April 29 3 May 30 16 June 29 520 July 28 709 August 27 419 ... to which of following cities? [CDS 2004] (a) Ahmedabad (b) Bangalore (c) Mumbai (d) Vadodara

Last Answer : Ans: (c)

Description : On what dates of April 1994 did SUNDAYfall? (A) 2,9,16,23,30 (B) 3,10,17,24 (C) 4,11,18,25 (D) 1,8,15,22,29

Last Answer : Answer: B Solution:. Let us find the day on 1st April, 1994. 2000 years have 0 odd day. We have to calculate day on 1st April, 1994. 2000-1994 = 6 Years. 1996 is leap year and have 4 odd days Ordinary years ... was Friday Thus, on 3 rd , 10 th , 17 th , 24 th will be Sunday fall on April 1994.

Description : If x^5 – 9x^2 + 12x – 14 is divisible by (x – 3), what is the remainder ? -Maths 9th

Last Answer : answer:

Description : What number are divisible by 14?

Last Answer : Itself and any multiples of 14

Description : The least number which should be added to 2497 so that the sum is exactly divisible by 5, 6, 4 and 3 is: A.10 B.14 C.23 D.30 E.None of these

Last Answer : Answer – C (23) Explanation – L.C.M. of 5, 6, 4 and 3 = 60. On dividing 2497 by 60, the remainder is 37. Number to be added = (60 – 37) = 23

Description : Which of the following represents the largest 4-digit number which can be added to 7249 in order to make the derived number divisible by each of 12,14,21,33 and 54? a) 9123 b) 9383 c) 8727 d) 8673

Last Answer : b) 9383

Description : What is the greatest 6-digit number that is divisible by 15, 24 and 28? a) 999650 b)999600 c)999825 d)999570

Last Answer : b)999600

Description : 20

Last Answer : 20 numbers from 1-100 are divisible by 5.

Description : What Number are divisible by 8 can be also divisible by what numbers?

Last Answer : They can also be evenly divided by 1, 2 and 4.

Description : What numbers are divisible by 4 or 6?

Last Answer : 12 and any multiples of 12

Description : What is 519 divisible by in whole numbers?

Last Answer : It is divisible by any of its factors which are 1, 3, 173 and519

Description : What which of these numbers is divisible by 12 is it a. 6 b. 26 c. 12 d. 42?

Last Answer : c.

Description : What which of these numbers are divisible by 13 is it a. 52 or b. 9 c. 30 d. 27?

Last Answer : a.

Description : What numbers are 21 divisible by?

Last Answer : They are members of the infinite set of numbers of the form 21*kwhere k is an integer.

Description : What numbers are divisible by 3 9 6?

Last Answer : Any number of the form 18*k where k is an integer.

Description : The least number that is divisible by all the numbers from 1 to 5 is: (a) 70 (b) 60 (c) 80 (d) 90

Last Answer : (b) 60

Description : For what value of m is x3 -2mx2 +16 divisible by x + 2 ? -Maths 9th

Last Answer : Let p(x) = x3 -2mx2 +16 Since, p(x) is divisible by (x+2), then remainder = 0 P(-2) = 0 ⇒ (-2)3 -2m(-2)2 + 16=0 ⇒ -8-8m+16=0 ⇒ 8 = 8 m m = 1 Hence, the value of m is 1 .

Description : For what value of m is x3 -2mx2 +16 divisible by x + 2 ? -Maths 9th

Last Answer : Let p(x) = x3 -2mx2 +16 Since, p(x) is divisible by (x+2), then remainder = 0 P(-2) = 0 ⇒ (-2)3 -2m(-2)2 + 16=0 ⇒ -8-8m+16=0 ⇒ 8 = 8 m m = 1 Hence, the value of m is 1 .

Description : For what value of k, will the expression (3x^3 – kx^2 + 4x + 16) be divisible by (x – k/2) ? -Maths 9th

Last Answer : Given f(x) = 3x³ - kx² + 4x + 16. Since (x - k/2) is a factor of polynomial. This means x = k/2 is the zero of the given polynomial. ⇒ f(k/2) = 3(k/2)³ - k(k/2)² + 4(k/2) + 16 ⇒ 0 ... - 4k + 32) + 4(k² - 4k + 32) ⇒ 0 = (k + 4)(k² - 4k + 32) ⇒ k = -4.

Description : The sum of 5 natural numbers A, B, C, D, E is 126. It is known that A:B = 7:8, B:C = 2:3, C:D = 4:5, D:E = 5:7.What is the difference between A and E? a) 14 b) 22 c) 24 d) 28 e) 32

Last Answer : A:B = 7:8 B:C = 2:3 = 8:12 SO, A:B:C = 7:8:12 C:D = 4:5 = 12:15 A:B:C:D = 7:8:12:15 D:E = 5:7 = 15:21 SO, A:B:C:D:E = 7:8:12:15:21 given x(7+8+12+15+21) = 126 or 63x = 126 or x =2 E – A = 21X – 7X = 14X = 28 Answer: d)

Description : In a telescope the object glass of focal length 14 cm, is located at 20 cm from the diaphragm. The focussing lens is midway between them when a staff 16.50 m away is focussed. The focal length of the focussing lens, is (A) 5.24 cm (B) 6.24 cm (C) 7.24 cm (D) 8.24 cm

Last Answer : (C) 7.24 cm

Description : 24 14 26 ? 28 16 30 a) 13 b) 17 c) 15 d) 11 e) 20

Last Answer : The series consists of two series 1 and 2: Series 1 : 24 26 28 30 +2 +2 +2 Series 2 : 14 15 16 +1 +1 Answer: c)

Description : 24 pumps are connected to a tank. Some of them are inlet pumps and the others are outlet pumps. Each of the inlet pumps an fill the tank in 16hours and each of the outlet pumps can empty the tank completely in ... gets filled in 48 hours. How many inlet pumps are there? A) 16 B) 18 C) 17 D) 14

Last Answer : D (x/16)-[(24-x)/12] = 1/48 x/16-24/48+x/12=1/48 (3+4)x/48=97/48 7x/48=97/48 7x=97 X=14

Description : A boat takes 24 hours to cover 128 km downstream and 16 hours to cover 64 km upstream. Then the speed of the boat in still water is: A) 14/3 B) 8/7 C) 3/2 D) 9/5

Last Answer : Answer: A Explanation: Distance covered in downstream = 128km Time taken in downstream = 24 hours. Rate of downstream = distance / time = a = 128 km /24 hours = 16/3km/hr Distance covered in upstream = 64km Time taken in upstream ... / 2 = (1/2)(16/3+4) km/hr = (1/2)(28/3)km/hr = 14/3km/hr.

Description : A two digit number exceeds forty percent of another two digit number by 16. If the sum of these two digit numbers is 72, which of the following is the difference between these two digit numbers ? a) 10 b) 6 c) 8 d) 14 e) None of these

Last Answer : c) 8 Let first two digit no. and second two digit no. be x and y respectively. x = y × 40/100 +16 x - 2y/5 = 16....(i) x + y = 72...(ii) From equation (i) and (ii) x = 32, y = 40 Difference = 8 Answer: c)

Description : The number of candidates writing three different entrance exams is in the ratio 4:5:6. There is a proposal to increase these numbers of candidates by 40%, 60% and 85% respectively. What will be the ratio of increased numbers? a) 14:15:16 b) 12:15:19 c)13:19:21 d) 14:16:19 E) None of these

Last Answer : Answer : E Given ratio of number of candidates is 4:5:6 Let the number of candidates for 3 exams be 4k, 5k and 6k respectively. After increasing, number of candidates become (140% of 4k), (160% of 5k) & (185% ... 111k/10 Now, the required new ratio is: 56k/100 : 80k/10 : 111k/10 = 56 : 80 : 111

Description : The largest numbers which divides 30, 78 and 102 to leave the same remainder in each case is (a)24 (b) 20 (c) 8 (d) 16

Last Answer : 24

Description : What is 14 x 29?

Last Answer : 406

Description : In a survery, it was found that `21` persons liked product `A`, `26` liked product `B` and `29` liked product `C`. If `14` persons liked products `A`

Last Answer : In a survery, it was found that `21` persons liked product `A`, `26` liked product `B` and `29` liked ... like the products `A` and `B` but not `C`

Description : A gas has a temperature of 14 degrees C, and a volume of 4.5 L. If the temperature is raised to 29 degrees C and the pressure is not changed, what is the new volume of the gas?

Last Answer : V1/T1 = V2/T2 (Charles' Law) T is in Kelvin. Celsius temp plus 273.15 = K T1 = 14 +273 = 287 K T2 = 29 + 273 = 303 K 4.5 / 287 = V2 / 303 V2 = 4.75 L = 4.8 L

Description : The optimum temperature for sugarcane is (a) 21-27 °C (b) 14-29 °C (c) 25-30 °C (d) 28-34 °C

Last Answer : (b) 14-29 °C

Description : The electric field intensity of a surface with permittivity 3.5 is given by 18 units. What the field intensity of the surface in air? a) 5.14 b) 0.194 c) 63 d) 29

Last Answer : c) 63

Description : 19/14 x 12/72 x 18/29 x 767 = ? (Take 2 digits after the decimal point) a) 110.69 b) 118.78 c) 111.56 d) None e) 108.98

Last Answer : 19/14 x 12/72 x 18/29 x 767 =? Sol: 1.35 x 0.16 x 0.62 x 767 = ? 102.71 = ? Answer: d)

Description : Area of a rectangle is equal to the area of circle whose radius is 14 cms. If the breadth of the rectangle is 22 cms. What is its length? a) 27 cms b) 28 cms c) 25 cms. d) 29 cms e) None of these

Last Answer : According to question, l×b = ¶r2 l ×22 = 22/7 ×14 × 14 l = 22/7 ×14 × 14/22 cms = 28 cms. Answer: b)

Description : The smallest number which when divided by 20, 25, 35 and 40 leaves a remainder of 14, 19 , 29 and 34 respectively is (a) 1994 (b) 1494 (c) 1394 (d) 1496

Last Answer : 1394 Hint: Note that 20 -4 = 6; 25 -19 = 6;35-29= 6;40- 34 = 6 Required number = L.C.M. of ( 20, 25, 35 and 40)-6

Description : The authority associated with the first genetically modified plants the soybean resistant to glyphosate from Monsanto which achieved widespread success was a.) Rachel Carson (May 27, 1907 - April 14, 1964) b). ... Fraley (birthdate: 1953) d). Norman Borlaug (March 25, 1914 - September 12, 2009)

Last Answer : c). Robert Fraley (birthdate: 1953)

Description : A national random sample of 20 ACT scores from 2010 is listed below. Calculate the sample mean and standard deviation. 29, 26, 13, 23, 23, 25, 17, 22, 17, 19, 12, 26, 30, 30, 18, 14, 12, 26, 17, 18 a. 20.50, 5.79 b. 20.50, 5.94 c. 20.85, 5.79 d. 20.85, 5.94

Last Answer : Katie earned 84,92,84,75 and 70 on her first 5 tests. What is the minimum grade Katie needs to earn on the next test to have a mean of 84?

Description : A ball falling vertically from rest for 3 seconds travels very nearly: w) 14.7 meters x) 29.4 meters y) 44.1 meters z) 88.2 meters

Last Answer : ANSWER: Y -- 44.1 METERS

Description : Let a means minus (–), b means multiplied by (ᵡ), C means divided by (÷) and D means plus (+). The value of 90 D 9 a 29 C 10 b 2 is (A) 8 (B) 10 (C) 12 (D) 14

Last Answer : Answer: *

Description : The average height of the students in a group was 360cm. When 10 students whose height is 292.8cm are newly admitted. The average height of the group was reduced by 24cm.How many students are present in the group? A) 29 B) 25 C) 28 D) 14

Last Answer : Answer: C) let the number of students initially in the class be A, then, total height = A * 360------ 1 again the no of students increased = A + 10 then, total height A + 10 student = (A +10) * 336 ... 3 (A+10) *336-2928 =A*360 336A+3360-2928=360A A=18 Number of students in class = 18+10=28

Description : What is/are the factors of (x^29 – x^24 + x^13 – 1) ? -Maths 9th

Last Answer : answer:

Description : The marks obtained by 15 students in an examination are given below : 40, 20, 24, 19,20, 35, 12, 48, 29, 40, 45, 48, 42, 23, 35. Find the average mark

Last Answer : The marks obtained by 15 students in an examination are given below : 40, 20, 24, 19,20, 35, 12, ... , 23, 35. Find the average mark of the students.